1
$\begingroup$

I am trying to understand the math. The inverse $\mathcal Z$-transform is given by:

$$x[n] = \displaystyle\frac{1}{j2\pi} \int_cX(z)z^{n-1}dz$$

where $\displaystyle \int_c$ is a contour integral. The inverse Fourier transform is given by:

$$x[n] = \displaystyle\frac{1}{2\pi}\int_{-\pi}^\pi X(e^{j\omega})e^{j\omega n}d\omega$$

My textbook claims, and I agree, that evaluating the inverse z-transform at $z=e^{j\omega}$ will result in the inverse Fourier transform. However, I can't get the math to show this. Substituting $z=e^{j\omega}$, I get as far as:

$$x[n] = \displaystyle \frac{1}{j2\pi}\int_cX(e^{j\omega})e^{j\omega n}e^{-j\omega}d(e^{j\omega})$$

Can somebody tell me intuitively how to simplify this equation to look like the inverse Fourier transform? Is my confusion due to a lack of understanding of the contour integral?

$\endgroup$

1 Answer 1

2
$\begingroup$

Hint:

With $z = e^{j\omega}$, you have:

\begin{align} \displaystyle \frac{dz}{d\omega} &= je^{j\omega}\\ \Rightarrow dz &= je^{j\omega} d\omega \end{align}

With $-\pi \leq \omega \leq \pi$, your contour integral goes from $-\pi$ to $\pi$.

Can somebody tell me intuitively how to simplify this equation to look like the inverse Fourier transform?

Plugging all the above should do the job.

Is my confusion due to a lack of understanding of the contour integral?

It is more the change of variables.

EDIT:

Note that $\omega$ is taken in the fundamental interval $\left[-\pi, \pi\right]$ because $X(\omega)$ is periodic with period $2\pi$. So, $\displaystyle \int_c$ becomes $\displaystyle \int_{2\pi}$

$\endgroup$
1
  • $\begingroup$ You're welcome. See the edit on the choice of $2\pi$ for the contour. $\endgroup$
    – Gilles
    Jul 24, 2015 at 23:04

Your Answer

By clicking “Post Your Answer”, you agree to our terms of service and acknowledge you have read our privacy policy.

Not the answer you're looking for? Browse other questions tagged or ask your own question.